0 Daumen
124 Aufrufe

Sei

\( A=\left(\begin{array}{lll} 3 & 0 & 1 \\ 0 & 1 & 2 \\ 0 & 3 & 4 \end{array}\right) . \)

Wir fassen \( A \) als Matrix in \( \mathbb{F}_{p} \) auf, wobei \( p \) eine beliebige Primzahl ist. Bestimmen Sie alle \( p \), für die \( A \) in \( M_{3}\left(\mathbb{F}_{p}\right) \) invertierbar ist.

Kann man bei dieser bzw. bei solchen Aufgaben einfach die Determinante bestimmen und dann det(A) mod p und falls det(A) mod p = 0 -> nicht invertierbar. Und sonst invertierbar.

Avatar von

Da hast du doch schon eine gute Idee beschrieben.

1 Antwort

0 Daumen

Dein Ansatz ist genau richtig.

Es gilt \(\det A=-6\), also ist \(\det A_2 \equiv_2 0\), \(\det A_3 \equiv_3 0\) und \(\det A_p \not\equiv_p 0\) für alle primen \(p\neq 2,3\). Wenn du jetzt begründest (oder die VL an der korrekten Stelle zitierst), wieso in \(\mathbb{F}_p\) auch Invertierbarkeit von Matrizen genau dann gegeben ist, wenn die Determinante nicht \(0\) ist, bist du fertig.

Avatar von

Ein anderes Problem?

Stell deine Frage

Ähnliche Fragen

2 Antworten
Gefragt 6 Okt 2016 von Gast
1 Antwort
Gefragt 2 Jul 2015 von alives
2 Antworten
Gefragt 23 Jun 2015 von Gast

Willkommen bei der Mathelounge! Stell deine Frage einfach und kostenlos

x
Made by a lovely community